Not Continuity of a multivariable function [duplicate]












0












$begingroup$



This question is an exact duplicate of:




  • Urgent with the continuity: Existence of directional derivatives at all directions of a $f:mathbb{R}^{2} to mathbb{R}$.

    1 answer




Let $f(x,y)=
begin{cases}
frac{2x^2y}{x^2+y^2} & text{if } (x,y) neq (0,0) \
0 & text{if } (x,y)=(0,0) .
end{cases}$



(i) Prove the directional derivatives of $f$ exist in any direction at the point $(0,0)$.



(ii)¿Is $f$ continuous on $(0,0)$?



For (i) I took $u=(u_{1},u_{2}) in mathbb{R^{2}}$ such $||u||=1$ and $0=(0,0)$. So $$lim_{t to 0} frac{f(0+ t u)-f(0)}{t}=lim_{t to 0}frac{2(tu_{1})^2 (tu_{2})}{(tu_{1})^4+(tu_{2})^2}=lim_{t to 0}frac{(t^2 u_{1}^2)(t^2 u_{1}^{2}) tu_{2}}{t^{4}u_{1}^{4}+t^{2} u_{2}^{2}}$$. But I cannot find the limit when $t$ aproaches to $0$. Which by finding them I would prove the directional derivative exist at every direction right?



For (ii) I got the intuition $f$ is not continuous at $(0,0)$ so I took $lbrace (frac{1}{k},frac{1}{k}) rbrace_{k in mathbb{N}}$ and $lbrace (frac{1}{k},0) rbrace_{k in mathbb{N}}$ which are two different sequences in $mathbb{R}^{2}$ converging to $(0,0)$. However, $lbrace f(frac{1}{k},frac{1}{k}) rbrace_{k in mathbb{N}}=lbrace frac{1}{k^{2}} rbrace_{n in mathbb{N}} to 0$ and $lbrace f(frac{1}{k},0) rbrace_{k in mathbb{N}} to 0$. So maybe my intuition was not right?



Also I've have approach $(0,0)$ through $(x,mx)$ and when this and gives me 0 as this value approaches to zero bit I cannot find another trajectory approaching trought $(0,0)$ which gives me another value distinct to $0$. :(



Can anyone help
me end the proof of continuity or not continuity , please?










share|cite|improve this question











$endgroup$



marked as duplicate by spaceisdarkgreen, KReiser, Jean-Claude Arbaut, Did, Jyrki Lahtonen Nov 26 '18 at 13:03


This question was marked as an exact duplicate of an existing question.




















    0












    $begingroup$



    This question is an exact duplicate of:




    • Urgent with the continuity: Existence of directional derivatives at all directions of a $f:mathbb{R}^{2} to mathbb{R}$.

      1 answer




    Let $f(x,y)=
    begin{cases}
    frac{2x^2y}{x^2+y^2} & text{if } (x,y) neq (0,0) \
    0 & text{if } (x,y)=(0,0) .
    end{cases}$



    (i) Prove the directional derivatives of $f$ exist in any direction at the point $(0,0)$.



    (ii)¿Is $f$ continuous on $(0,0)$?



    For (i) I took $u=(u_{1},u_{2}) in mathbb{R^{2}}$ such $||u||=1$ and $0=(0,0)$. So $$lim_{t to 0} frac{f(0+ t u)-f(0)}{t}=lim_{t to 0}frac{2(tu_{1})^2 (tu_{2})}{(tu_{1})^4+(tu_{2})^2}=lim_{t to 0}frac{(t^2 u_{1}^2)(t^2 u_{1}^{2}) tu_{2}}{t^{4}u_{1}^{4}+t^{2} u_{2}^{2}}$$. But I cannot find the limit when $t$ aproaches to $0$. Which by finding them I would prove the directional derivative exist at every direction right?



    For (ii) I got the intuition $f$ is not continuous at $(0,0)$ so I took $lbrace (frac{1}{k},frac{1}{k}) rbrace_{k in mathbb{N}}$ and $lbrace (frac{1}{k},0) rbrace_{k in mathbb{N}}$ which are two different sequences in $mathbb{R}^{2}$ converging to $(0,0)$. However, $lbrace f(frac{1}{k},frac{1}{k}) rbrace_{k in mathbb{N}}=lbrace frac{1}{k^{2}} rbrace_{n in mathbb{N}} to 0$ and $lbrace f(frac{1}{k},0) rbrace_{k in mathbb{N}} to 0$. So maybe my intuition was not right?



    Also I've have approach $(0,0)$ through $(x,mx)$ and when this and gives me 0 as this value approaches to zero bit I cannot find another trajectory approaching trought $(0,0)$ which gives me another value distinct to $0$. :(



    Can anyone help
    me end the proof of continuity or not continuity , please?










    share|cite|improve this question











    $endgroup$



    marked as duplicate by spaceisdarkgreen, KReiser, Jean-Claude Arbaut, Did, Jyrki Lahtonen Nov 26 '18 at 13:03


    This question was marked as an exact duplicate of an existing question.


















      0












      0








      0





      $begingroup$



      This question is an exact duplicate of:




      • Urgent with the continuity: Existence of directional derivatives at all directions of a $f:mathbb{R}^{2} to mathbb{R}$.

        1 answer




      Let $f(x,y)=
      begin{cases}
      frac{2x^2y}{x^2+y^2} & text{if } (x,y) neq (0,0) \
      0 & text{if } (x,y)=(0,0) .
      end{cases}$



      (i) Prove the directional derivatives of $f$ exist in any direction at the point $(0,0)$.



      (ii)¿Is $f$ continuous on $(0,0)$?



      For (i) I took $u=(u_{1},u_{2}) in mathbb{R^{2}}$ such $||u||=1$ and $0=(0,0)$. So $$lim_{t to 0} frac{f(0+ t u)-f(0)}{t}=lim_{t to 0}frac{2(tu_{1})^2 (tu_{2})}{(tu_{1})^4+(tu_{2})^2}=lim_{t to 0}frac{(t^2 u_{1}^2)(t^2 u_{1}^{2}) tu_{2}}{t^{4}u_{1}^{4}+t^{2} u_{2}^{2}}$$. But I cannot find the limit when $t$ aproaches to $0$. Which by finding them I would prove the directional derivative exist at every direction right?



      For (ii) I got the intuition $f$ is not continuous at $(0,0)$ so I took $lbrace (frac{1}{k},frac{1}{k}) rbrace_{k in mathbb{N}}$ and $lbrace (frac{1}{k},0) rbrace_{k in mathbb{N}}$ which are two different sequences in $mathbb{R}^{2}$ converging to $(0,0)$. However, $lbrace f(frac{1}{k},frac{1}{k}) rbrace_{k in mathbb{N}}=lbrace frac{1}{k^{2}} rbrace_{n in mathbb{N}} to 0$ and $lbrace f(frac{1}{k},0) rbrace_{k in mathbb{N}} to 0$. So maybe my intuition was not right?



      Also I've have approach $(0,0)$ through $(x,mx)$ and when this and gives me 0 as this value approaches to zero bit I cannot find another trajectory approaching trought $(0,0)$ which gives me another value distinct to $0$. :(



      Can anyone help
      me end the proof of continuity or not continuity , please?










      share|cite|improve this question











      $endgroup$





      This question is an exact duplicate of:




      • Urgent with the continuity: Existence of directional derivatives at all directions of a $f:mathbb{R}^{2} to mathbb{R}$.

        1 answer




      Let $f(x,y)=
      begin{cases}
      frac{2x^2y}{x^2+y^2} & text{if } (x,y) neq (0,0) \
      0 & text{if } (x,y)=(0,0) .
      end{cases}$



      (i) Prove the directional derivatives of $f$ exist in any direction at the point $(0,0)$.



      (ii)¿Is $f$ continuous on $(0,0)$?



      For (i) I took $u=(u_{1},u_{2}) in mathbb{R^{2}}$ such $||u||=1$ and $0=(0,0)$. So $$lim_{t to 0} frac{f(0+ t u)-f(0)}{t}=lim_{t to 0}frac{2(tu_{1})^2 (tu_{2})}{(tu_{1})^4+(tu_{2})^2}=lim_{t to 0}frac{(t^2 u_{1}^2)(t^2 u_{1}^{2}) tu_{2}}{t^{4}u_{1}^{4}+t^{2} u_{2}^{2}}$$. But I cannot find the limit when $t$ aproaches to $0$. Which by finding them I would prove the directional derivative exist at every direction right?



      For (ii) I got the intuition $f$ is not continuous at $(0,0)$ so I took $lbrace (frac{1}{k},frac{1}{k}) rbrace_{k in mathbb{N}}$ and $lbrace (frac{1}{k},0) rbrace_{k in mathbb{N}}$ which are two different sequences in $mathbb{R}^{2}$ converging to $(0,0)$. However, $lbrace f(frac{1}{k},frac{1}{k}) rbrace_{k in mathbb{N}}=lbrace frac{1}{k^{2}} rbrace_{n in mathbb{N}} to 0$ and $lbrace f(frac{1}{k},0) rbrace_{k in mathbb{N}} to 0$. So maybe my intuition was not right?



      Also I've have approach $(0,0)$ through $(x,mx)$ and when this and gives me 0 as this value approaches to zero bit I cannot find another trajectory approaching trought $(0,0)$ which gives me another value distinct to $0$. :(



      Can anyone help
      me end the proof of continuity or not continuity , please?





      This question is an exact duplicate of:




      • Urgent with the continuity: Existence of directional derivatives at all directions of a $f:mathbb{R}^{2} to mathbb{R}$.

        1 answer








      multivariable-calculus continuity






      share|cite|improve this question















      share|cite|improve this question













      share|cite|improve this question




      share|cite|improve this question








      edited Nov 26 '18 at 9:17









      Tianlalu

      3,08621038




      3,08621038










      asked Nov 26 '18 at 3:44









      CosCos

      1326




      1326




      marked as duplicate by spaceisdarkgreen, KReiser, Jean-Claude Arbaut, Did, Jyrki Lahtonen Nov 26 '18 at 13:03


      This question was marked as an exact duplicate of an existing question.






      marked as duplicate by spaceisdarkgreen, KReiser, Jean-Claude Arbaut, Did, Jyrki Lahtonen Nov 26 '18 at 13:03


      This question was marked as an exact duplicate of an existing question.
























          2 Answers
          2






          active

          oldest

          votes


















          1












          $begingroup$

          Using polar coordinates,
          $midfrac{2x^2y}{x^2+y^2}mid=midfrac{2r^2cos^2theta rsintheta}{r^2cos^2theta +r^2sin^2theta}mid=midfrac{2r^3cos^2thetasintheta}{r^2}midle 2rto0$ as $rto0$.



          Hence $f$ is continuous at the origin.






          share|cite|improve this answer











          $endgroup$





















            0












            $begingroup$

            Using AM-GM inequality, x^2+y^2 >= 2xy. 1/(x^2+y^2) =< 1/2xy.



            So |2x^2y/(x^2+y^2)| =< |(2x^2y/2xy = x )| which approaches to zero as x approaches to zero.



            Hence it is continuous at (0,0).






            share|cite|improve this answer









            $endgroup$













            • $begingroup$
              Hi and welcome to math.SE. Please use MathJax formatting to improve readability of your answer.
              $endgroup$
              – francescop21
              Nov 26 '18 at 12:02


















            2 Answers
            2






            active

            oldest

            votes








            2 Answers
            2






            active

            oldest

            votes









            active

            oldest

            votes






            active

            oldest

            votes









            1












            $begingroup$

            Using polar coordinates,
            $midfrac{2x^2y}{x^2+y^2}mid=midfrac{2r^2cos^2theta rsintheta}{r^2cos^2theta +r^2sin^2theta}mid=midfrac{2r^3cos^2thetasintheta}{r^2}midle 2rto0$ as $rto0$.



            Hence $f$ is continuous at the origin.






            share|cite|improve this answer











            $endgroup$


















              1












              $begingroup$

              Using polar coordinates,
              $midfrac{2x^2y}{x^2+y^2}mid=midfrac{2r^2cos^2theta rsintheta}{r^2cos^2theta +r^2sin^2theta}mid=midfrac{2r^3cos^2thetasintheta}{r^2}midle 2rto0$ as $rto0$.



              Hence $f$ is continuous at the origin.






              share|cite|improve this answer











              $endgroup$
















                1












                1








                1





                $begingroup$

                Using polar coordinates,
                $midfrac{2x^2y}{x^2+y^2}mid=midfrac{2r^2cos^2theta rsintheta}{r^2cos^2theta +r^2sin^2theta}mid=midfrac{2r^3cos^2thetasintheta}{r^2}midle 2rto0$ as $rto0$.



                Hence $f$ is continuous at the origin.






                share|cite|improve this answer











                $endgroup$



                Using polar coordinates,
                $midfrac{2x^2y}{x^2+y^2}mid=midfrac{2r^2cos^2theta rsintheta}{r^2cos^2theta +r^2sin^2theta}mid=midfrac{2r^3cos^2thetasintheta}{r^2}midle 2rto0$ as $rto0$.



                Hence $f$ is continuous at the origin.







                share|cite|improve this answer














                share|cite|improve this answer



                share|cite|improve this answer








                edited Nov 26 '18 at 5:36

























                answered Nov 26 '18 at 5:13









                Chris CusterChris Custer

                11.4k3824




                11.4k3824























                    0












                    $begingroup$

                    Using AM-GM inequality, x^2+y^2 >= 2xy. 1/(x^2+y^2) =< 1/2xy.



                    So |2x^2y/(x^2+y^2)| =< |(2x^2y/2xy = x )| which approaches to zero as x approaches to zero.



                    Hence it is continuous at (0,0).






                    share|cite|improve this answer









                    $endgroup$













                    • $begingroup$
                      Hi and welcome to math.SE. Please use MathJax formatting to improve readability of your answer.
                      $endgroup$
                      – francescop21
                      Nov 26 '18 at 12:02
















                    0












                    $begingroup$

                    Using AM-GM inequality, x^2+y^2 >= 2xy. 1/(x^2+y^2) =< 1/2xy.



                    So |2x^2y/(x^2+y^2)| =< |(2x^2y/2xy = x )| which approaches to zero as x approaches to zero.



                    Hence it is continuous at (0,0).






                    share|cite|improve this answer









                    $endgroup$













                    • $begingroup$
                      Hi and welcome to math.SE. Please use MathJax formatting to improve readability of your answer.
                      $endgroup$
                      – francescop21
                      Nov 26 '18 at 12:02














                    0












                    0








                    0





                    $begingroup$

                    Using AM-GM inequality, x^2+y^2 >= 2xy. 1/(x^2+y^2) =< 1/2xy.



                    So |2x^2y/(x^2+y^2)| =< |(2x^2y/2xy = x )| which approaches to zero as x approaches to zero.



                    Hence it is continuous at (0,0).






                    share|cite|improve this answer









                    $endgroup$



                    Using AM-GM inequality, x^2+y^2 >= 2xy. 1/(x^2+y^2) =< 1/2xy.



                    So |2x^2y/(x^2+y^2)| =< |(2x^2y/2xy = x )| which approaches to zero as x approaches to zero.



                    Hence it is continuous at (0,0).







                    share|cite|improve this answer












                    share|cite|improve this answer



                    share|cite|improve this answer










                    answered Nov 26 '18 at 11:57









                    SouvikSouvik

                    1




                    1












                    • $begingroup$
                      Hi and welcome to math.SE. Please use MathJax formatting to improve readability of your answer.
                      $endgroup$
                      – francescop21
                      Nov 26 '18 at 12:02


















                    • $begingroup$
                      Hi and welcome to math.SE. Please use MathJax formatting to improve readability of your answer.
                      $endgroup$
                      – francescop21
                      Nov 26 '18 at 12:02
















                    $begingroup$
                    Hi and welcome to math.SE. Please use MathJax formatting to improve readability of your answer.
                    $endgroup$
                    – francescop21
                    Nov 26 '18 at 12:02




                    $begingroup$
                    Hi and welcome to math.SE. Please use MathJax formatting to improve readability of your answer.
                    $endgroup$
                    – francescop21
                    Nov 26 '18 at 12:02



                    Popular posts from this blog

                    Biblatex bibliography style without URLs when DOI exists (in Overleaf with Zotero bibliography)

                    ComboBox Display Member on multiple fields

                    Is it possible to collect Nectar points via Trainline?